Đến nội dung

minhtuyb nội dung

Có 497 mục bởi minhtuyb (Tìm giới hạn từ 06-06-2020)



Sắp theo                Sắp xếp  

#371298 BĐT AM-GM

Đã gửi bởi minhtuyb on 21-11-2012 - 20:16 trong Bất đẳng thức và cực trị

3,(tổng quát bài 1 lên) với 2n số không âm
$\sqrt[n]{a_{1}a_{2}....a_{n}} +\sqrt[n]{b_{1}b_{2}...b_{n}}\leq \sqrt[n]{(a_{1}+b_{1})(a_{2}+b_{2})...(a_{n}+b_{n})}\(*)$

Nhìn phát thấy ngay là $Holder$ Hình đã gửi
---
-Nếu $\exists a_i+b_i=0\Rightarrow a_i=b_i=0\Rightarrow VT(*)=VP(*)$
-Nếu các biến đều dương: áp dụng BĐT $AM-GM$ cho $n$ số, ta có:
$$\dfrac{a_1}{a_1+b_1}+\dfrac{a_2}{a_2+b_2}+...+\dfrac{a_n}{a_n+b_n}\ge \dfrac{n\sqrt[n]{a_1a_2...a_n}}{(a_{1}+b_{1})(a_{2}+b_{2})...(a_{n}+b_{n})}\\
\dfrac{b_1}{a_1+b_1}+\dfrac{b_2}{a_2+b_2}+...+\dfrac{b_n}{a_n+b_n}\ge \dfrac{n\sqrt[n]{b_1b_2...b_n}}{(a_{1}+b_{1})(a_{2}+b_{2})...(a_{n}+b_{n})}$$
Cộng vế với vế của 2 BĐT cùng chiều trên, sau vài bước biến đổi ta có ĐPCM. Dấu bằng xảy ra chẳng hạn khi $a_1=a_2=...=a_n;b_1=b_2=...=b_n\ \square$
---
Hai bài trên là hệ quả trực tiếp của BĐT này



#371072 Chứng minh tỉ số GA/GF và PA/PG không đổi khi P di chuyển

Đã gửi bởi minhtuyb on 20-11-2012 - 21:24 trong Hình học phẳng

cho tam giác ABC,P ,bất kì.D,E,F là điểm đối xứng với P qua BC,CA,AB.BD cắt EF tại G.chứng minh tỉ số GA/GF và PA/PG không đổi khi P di chuyển( GA,GF,PA,PG là độ dài đại số nha,t ko viết gạch trên đầu)

Chú chép đề kiểu gì thế, đề anh chép là như thế này:
$\Delta ABC, P$ bất kì. $D,E,F$ là điểm đối xứng với $P$ qua $BC,CA,AB$. $PD\cap EF\equiv G$.
CMR: $\dfrac{\overline{GE}}{\overline{GF}};\dfrac{\overline{PD}}{\overline{PG}}$ không đổi khi $P$ di chuyển



#371030 Chứng minh rằng $x^2-n$ không là số chính phương

Đã gửi bởi minhtuyb on 20-11-2012 - 20:15 trong Các dạng toán THPT khác

Giả sử $n\in \mathbb{N^*} ,p$ là ước nguyên tố nhỏ nhất của $n$ và $ x \in \mathbb{N}$ thỏa mãn:$x>\dfrac{n+p^2}{2p}$.
Chứng minh rằng $x^2-n$ không là số chính phương
___
NLT: Học cách gõ công thức tại đây !

Giúp người giúp cho chót Hình đã gửi
------
-Từ giả thiết suy ra $n<2px-p^2$. Từ đây ta kẹp được:
$$(x-p)^2<x^2-n<x^2$$
Vậy để $x^2-n$ là số chính phương thì phải $\exists k\ (0<k<p)\ (*)$ sao cho:
$$x^2-n=(x-p+k)^2\\ \Leftrightarrow x^2-(x-p+k)^2=n\\ \Leftrightarrow (p-k)(2x-p+k)=n\ (1)$$
Vì $n\ne 0$ nên từ $(1)$ suy ra: $n\vdots p-k$. Lại có $n\vdots p$ theo giả thiết $\Rightarrow n\vdots k\ (2)$.
Mà do $p$ là ước nguyên tố nhỏ nhất của $n$ nên từ $(2)$ suy ra $k\ge p\ (**)$
-Từ $(*)$ với $(**)$ dẫn đến mâu thuẫn. Vậy $x^2-n$ không phải số chính phương $\square$

---
Hôm nào thầy gọi anh chú cũng phải giúp nhé Hình đã gửi



#371017 Chứng minh rằng có vô hạn các số có dạng $a_n=2^n-3$ ($n...

Đã gửi bởi minhtuyb on 20-11-2012 - 19:46 trong Số học

Chứng minh rằng có vô hạn các số có dạng $a_n=2^n-3$ ($n \geq 2$) đôi một nguyên tố cùng nhau

Xét : $n>1$ thì $a_n$ lẻ. Ta có hai giá trị khởi đầu:
$$a_2=1;a_3=5\Rightarrow (a_2;a_3)=1$$

Ta sẽ chứng minh: Giả sử ta chọn được dãy $b_1,b_2,...,b_k$ là dãy con của dãy $a_n$ thỏa mãn các phần tử của dãy đôi một nguyên tố cùng nhau. Ta sẽ chứng minh $\exists b_{k+1}$ đôi một nguyên tố cùng nhau với $b_1,b_2,...,b_k$. Thật vậy, đặt:
$$A=b_1.b_2....b_k\\ \Rightarrow (A,2)=1$$
Áp dụng định lí Euler:
$$2^{\varphi(A)}-1\vdots A$$
Mà $A$ lẻ $\Rightarrow (2^{\varphi(A)}-3,A)=1\Rightarrow 2^{\varphi(A)}-3$ đôi một nguyên tố cùng nhau với $b_1,b_2,...,b_k$.
Vậy ta chọn $b_{k+1}=\varphi(A)$.
Ta đã c/m được sự vô hạn của dãy $b_k$. Vậy mệnh đề được c/m $\square$



#370878 $cos^{4}\widehat{MOA}+cos^{4}\wi...

Đã gửi bởi minhtuyb on 20-11-2012 - 11:26 trong Hình học phẳng

Cho $\bigtriangleup ABC$ đều nội tiếp (o). M $M\epsilon$ (o) a)
cmr a) cos$\widehat{MOA}$ +cos$\widehat{MOB}$ +cos$\widehat{MOC}$=0 ( da chung minh duoc)
b)
$cos^{2}\widehat{MOA}+cos^{2}\widehat{MOB}+cos^{2}\widehat{MOC}=\frac{3}{2}$
c)
$cos^{4}\widehat{MOA}+cos^{4}\widehat{MOB}+cos^{4}\widehat{MOC}=\frac{9}{8}$

Bài c/m khá dài nên mình chỉ nói vài ý chính:
Giả sử $M\in $ cung nhỏ $BC$. Ta sẽ c/m lần lượt:
+)$MA^2+MB^2+MC^2=6R^2$ (chú ý: $R^2=OA^2=(\overrightarrow{MA}-\overrightarrow{MO})^2=...$)
+)$MA^4+MB^4+MC^4=2a^4$ với $a$ là cạnh của tam giác, sau đó tính $a$ theo $R$.
(Chú ý $MA=MB+MC;MB^2+MC^2+MB.MC=a^2$)

Cuối cùng là ráp lại theo công thức:
$\sum cos^{2}\widehat{MOA}=\sum \left(\dfrac{\overrightarrow{OM}.\overrightarrow{OA}}{OM.OA} \right)^2=\sum \left(\dfrac{OM^2+OA^2-AM^2}{2OM.OA} \right)^2$
Sau đó biến đổi trâu sẽ ra. :P
---
Phần c thực chất là một bài toán biến đổi đại số :
Cho $\left\{\begin{matrix}x+y+z=0\\ x^2+y^2+z^2=\dfrac{3}{2}\end{matrix}\right.$
CMR: $x^4+y^4+z^4=\dfrac{9}{8}$



#370846 Đếm số hình vuông $n*n$ sao cho mỗi hàng, mỗi cột có tổng các số là...

Đã gửi bởi minhtuyb on 20-11-2012 - 10:33 trong Tổ hợp và rời rạc

Problem: Đếm số hình vuông $n*n$, trong đó các ô vuông con $1*1$ được điền các số $0,1$ sao cho mỗi hàng, mỗi cột có tổng các số là chẵn.



#370841 Đề thi học sinh giỏi tỉnh môn toán lớp 10 tỉnh Hà Tĩnh năm học 2011-2012

Đã gửi bởi minhtuyb on 20-11-2012 - 10:24 trong Thi HSG cấp Tỉnh, Thành phố. Olympic 30-4. Đề thi và kiểm tra đội tuyển các cấp.

Câu 4: Các số thực dương $x,y,z$ thỏa mãn điều kiện $x+y+z=\frac{3}{2}$
Tìm giá trị nhỏ nhất của biểu thức:
$$M=\frac{\sqrt{x^2+xy+y^2}}{4yz+1}+\frac{\sqrt{y^2+yz+z^2}}{4zx+1}+\frac{\sqrt{z^2+zx+x^2}}{4xy+1}$$


OTHER SOLUTION:
Ta cũng c/m $M.\dfrac{2}{\sqrt{3}}\ge \frac{x+y}{4yz+1}+\frac{y+z}{4zx+1}+\frac{z+x}{4xy+1}$ như trên.
Áp dụng BĐT Schwarz và bổ đề $x^2y+y^2z+z^2x+xyz\le \dfrac{4}{27} (x+y+z)^3$, ta có:
$$M.\dfrac{2}{\sqrt{3}}\ge \sum \dfrac{(x+y)^2}{4yz(x+y)+x+y}\ge \dfrac{4(x+y+z)^2}{12xyz+4(x^2y+y^2z+z^2x)+2(x+y+z)} =\dfrac{9}{4(x^2y+y^2z+z^2x+xyz)+8xyz+3}\\ \ge \dfrac{9}{4.\dfrac{4}{27}(x+y+z)^3+8.\dfrac{(x+y+z)^3}{27}+3}=\dfrac{3}{2}\\ \Rightarrow M\ge \dfrac{3\sqrt{3}}{4}$$
Dấu bằng xảy ra khi $x=y=z=\dfrac{1}{2}$
Vậy $minM=\dfrac{3\sqrt{3}}{4}$ khi $x=y=z=\dfrac{1}{2}\ \square$
---
Mình thấy cách này ý tưởng có vẻ tự nhiên hơn :D



#370836 Đề thi chọn học sinh giỏi khối 12 THPT Chuyên Toán Thừa Thiên Huế 2012 - 2013

Đã gửi bởi minhtuyb on 20-11-2012 - 10:05 trong Thi HSG cấp Tỉnh, Thành phố. Olympic 30-4. Đề thi và kiểm tra đội tuyển các cấp.

Bài 4 : (4 điểm)
Cho tam giác nhọn $ABC$ có $AB<BC<CA$. Đường tròn nội tiếp của tam giác $ABC$ có tâm $I$ và lần lượt tiếp xúc với các cạnh $BC, CA, AB$ tại $D, E, F$.
Chứng minh rằng các đường tròn ngoại tiếp của các tam giác $IAD, IBE, ICF$ có tâm cùng nằm trên một đường thẳng.

Gọi $A_1,B_1,C_1$ làn lượt là trung điểm của $EF,FD,DE$.Ta có $DA_1,EB_1,FC_1$ đồng quy tại trọng tâm $G$ của $\Delta DEF$.
Xét phép nghịch đảo tâm $I$ phương tích $k=r^2$,ta có: $A_1 \rightarrow A, D \rightarrow D \Rightarrow A_1D \rightarrow (IAD).$Tương tự $EB_1 \rightarrow (IBE); FC_1 \rightarrow (ICF).$
Vì $DA_1,EB_1,FC_1$ đồng quy tại 1 điểm nên 3 đường tròn $(IAD),(IBE),(ICF)$ cũng đồng quy tại một điểm khác $I$ hay chúng có chung trục đẳng phương,suy ra tâm 3 đường tròn trên thẳng hàng.
---
Nguồn: MS



#370809 ĐỀ THI LẬP ĐỘI TUYỂN HỌC SINH GIỎI TOÁN DỰ THI QUỐC GIA TỈNH ĐAKLAK NĂM 2012-...

Đã gửi bởi minhtuyb on 20-11-2012 - 00:00 trong Thi HSG cấp Tỉnh, Thành phố. Olympic 30-4. Đề thi và kiểm tra đội tuyển các cấp.

Ta có $x_1;x_2;x_3$ là 3 nghiệm của phương trình $x^{3}+ax^{2}+x+b=0$.
Áp dụng định lý $Vi-et$ bậc 3 ta có: $x_1x_2+x_1x_3+x_2x_3=1$
Áp dụng bất đẳng thức $Cauchy-Schwarz$ và bất đẳng thức quen thuộc $a^2+b^2+c^2\ge ab+bc+ac$ ta có :
$$ \dfrac{x_1^3}{x_2+x_3}+\dfrac{x_2^3}{x_1+x_2}+\dfrac{x_3^3}{x_1+x_2}=\dfrac{x_1^4}{x_1x_2+x_1x_3}+\dfrac{x_2^4}{x_2x_1+x_3}+\dfrac{x_3^2}{x_3x_1+x_3x_2}$$
$$\geq \dfrac{(x_1^2+x_2^2+x_3^2)^2}{2(x_1x_2+x_2x_3+x_3x_1)}\ge \dfrac{(x_1x_2+x_2x_3+x_1x_3)^2}{2(x_2x_1+x_1x_3+x_2x_3)}=\dfrac{x_2x_1+x_1x_3+x_2x_3}{2}=\dfrac{1}{2}$$
Dấu "=" không xảy ra do đó ta có điều cần chứng minh.

Đoạn dùng Schwarz có chắc là mấy cái dưới mẫu không âm không cậu?



#370808 ĐỀ THI LẬP ĐỘI TUYỂN HỌC SINH GIỎI TOÁN DỰ THI QUỐC GIA TỈNH ĐAKLAK NĂM 2012-...

Đã gửi bởi minhtuyb on 19-11-2012 - 23:57 trong Thi HSG cấp Tỉnh, Thành phố. Olympic 30-4. Đề thi và kiểm tra đội tuyển các cấp.

Ta có:
$u_{n+1}-u_n=(n+1)(u_n-u_{n-1})$
hay $u_n-u_{n-1}=n(u_{n-1}-u_{n-2})=n(n-1)(u_{n-2}-u_{n-3}) =...= $
$=n(n-1)...3(u_2-u_1)=n!$
Suy ra: $u_n=u_{n-1}+n!=u_{n-2}+(n-1)!+n!=...=1!+2!+...+n!$

Đến đây thì phải làm thế nào nữa nhỉ? :wacko:

Em xin hoàn thiện nốt lời giải :D
-----
Giả sử $u_n=a^p\ \ (a,p\ge 2)$
Xét 2 trường hợp:

TH 1: Nếu $p=2$ thì ta đưa về ngay pt nghiệm nguyên quen thuộc:
$$1!+2!+...+n!=a^2$$
Dễ dàng c/m pt vô nghiệm với $n>8$ ($VT$ có chữ số tận cùng là $3$)
Vậy $u_n$ không phải SCP

TH 2: Nếu $p\ge 3$: Với mọi $n\ge 2$ thì:
$$u_n=1!+2!+\sum\limits_{k=3}^{n} k! \vdots 3$$
Vì $u_n\vdots 3,p\ge 3$ nên $u_n\vdots 27\ \forall n\ge 2\ \ (*)$

-Xét $n\ge 9$:
$$u_n=u_8+\sum\limits_{k=9}^{n} k! $$
Vì $k!\vdots 27\ \forall k\ge 9\Rightarrow u_n\equiv u_8\ (\mod 27)\ \forall n\ge 9\ (1)$
Mặt khác, bằng tính toán trực tiếp ta thu được:
$$u_8=1!+2!+...+8!=46233\not{\vdots}\; 27\ (2)$$
-Từ $(1),(2)\Rightarrow u_n\not{\vdots}\; 27\forall n\ge 9\ (**)$
Từ $(*)$ và $(**)$ ta có mâu thuẫn. Vậy trong trường hợp này $u_n$ không thể biểu diễn dưới dạng lũy thừa với số mũ nguyên dương của một số tự nhiên lớn hơn hoặc bằng 3.

Kết hợp 2 trường hợp trên ta có ĐPCM $\square$
----
Bài này mà không cho mang máy tính vào thì ... :P



#370281 CMR: $\sqrt{x^2+(y-1)^2}+\sqrt{x^2+(y+1)^2...

Đã gửi bởi minhtuyb on 18-11-2012 - 09:06 trong Bất đẳng thức và cực trị

bạn giải tiếp đi
mình áp dụng rồi mà không ra

Cho x,y thuộc [-1;1]. CMR:
$\sqrt{x^2+(y-1)^2}+\sqrt{x^2+(y+1)^2}+\sqrt{(x-1)^2+y^2}+\sqrt{(x+1)^2+y^2}\geq 4$

Sr nhìn không kĩ, Min chỉ c/m được $\ge 2\sqrt{2}$...
----

Tổng quát $x,y\in \mathbb{R}$ thì BĐT vẫn đúng:
Dự đoán điểm rơi ở $x=y=0$ nên ta có các đánh giá sau:
$$\sqrt{x^2+(y-1)^2}\ge \sqrt{(y-1)^2}=|1-y|\\ \sqrt{x^2+(y+1)^2}\ge \sqrt{(y+1)^2}=|y+1|\\ \sqrt{(x-1)^2+y^2}\ge \sqrt{(x-1)^2}=|1-x|\\\sqrt{(x+1)^2+y^2}\ge \sqrt{(x+1)^2}=|x+1|$$
Từ các điều trên suy ra:
$$LHS\ge |1-y|+|y+1|+|1-x|+|x+1|\ge |1-y+y+1+1-x+x+1|=4=RHS$$
Dấu bằng xảy ra khi $x=y=0\ \square$
----
P/s: Cái giả thiết làm được gì cho đời nhỉ?



#370243 Tìm min của (a+b+c)/(b-a)

Đã gửi bởi minhtuyb on 17-11-2012 - 23:06 trong Bất đẳng thức và cực trị

Gợi ý: $f(-2)\ge 0$...
TQ: xét tam thức bậc 2 $f(x)=ax^2+bx+c$ ($a \ne 0;ma>-b$), $f(x)\ge 0\forall x$
Tìm Min của
$$M=\dfrac{a+b+c}{ma+b}$$



#370185 giải toán tổ hợp dùng phép song ánh

Đã gửi bởi minhtuyb on 17-11-2012 - 21:40 trong Tổ hợp và rời rạc

Bài 1 tham khảo thêm ở http://diendantoanho...ia-cac-dội-bơi/



#369536 Topic về các bài toán: Tập hợp - Logic tập hợp

Đã gửi bởi minhtuyb on 14-11-2012 - 23:09 trong Tổ hợp và rời rạc

Problem 3: Có bao nhiêu cách chọn ba tập con $A,B,C$ của $\left\{ 1,2,...,n\right\}$ thỏa $A\subseteq C;B\subseteq C;A\cap B\neq\varnothing$



#369527 Topic về các bài toán: Tập hợp - Logic tập hợp

Đã gửi bởi minhtuyb on 14-11-2012 - 22:32 trong Tổ hợp và rời rạc

-Xét $A,B$ là hai tập con thỏa mãn đề bài.
-Với mỗi phần tử trong tập hợp $X$, có 3 khả năng xảy ra:
+) $x\in A;x\not\in B$
+) $x\in B;x\not\in A$
+) $x\not\in A;x\not\in B$
Vì tập $X$ có $n$ phần tử $\Rightarrow $ có $3^n$ trạng thái $\Rightarrow$ có tổng cộng $3^n$ cặp tập con thỏa mãn đề bài.
-------------------------------------------------------------
Đây là topic thảo luận, không cần xoá đâu em!

Em quên đếm số tập bị lặp :P:
----
Bổ sung: Trong $3^n$ cặp tập con đó có một cặp $(\varnothing
;\varnothing)$ và các cặp còn lại, mỗi cặp được đếm 2 lần nên số tập con thỏa mãn là: $\dfrac{3^n-1}{2}+1=\dfrac{3^n+1}{2}$



#369520 Topic về các bài toán: Tập hợp - Logic tập hợp

Đã gửi bởi minhtuyb on 14-11-2012 - 22:08 trong Tổ hợp và rời rạc

Có gì thắc mắc mọi người có thể thắc mắc ngay trong topic này, đề bài nhớ ghi rõ nguồn nếu biết, mong mọi người ủng hộ. Bây giờ ta vào vấn đề chính nào:
Problem 1: Có bao nhiêu cặp tập con không giao nhau của một tập hợp $X$ gồm $n$ phần tử.

-Xét $A,B$ là hai tập con thỏa mãn đề bài.
-Với mỗi phần tử trong tập hợp $X$, có 3 khả năng xảy ra:
+) $x\in A;x\not\in B$
+) $x\in B;x\not\in A$
+) $x\not\in A;x\not\in B$
Vì tập $X$ có $n$ phần tử $\Rightarrow $ có $3^n$ trạng thái $\Rightarrow$ có tổng cộng $3^n$ cặp tập con thỏa mãn đề bài.
-------------------------------------------------------------
Đây là topic thảo luận, không cần xoá đâu em!



#368892 Tìm hàm số $f(x):R \rightarrow R$ thỏa mãn: $f(f(x)) = f...

Đã gửi bởi minhtuyb on 11-11-2012 - 23:09 trong Đại số

Bài này theo mình ta còn phải chứng minh $f(x)$ là một hàm số tuyến tính bởi tính chất tuyến tính của nó không quá hiển nhiên
Theo m có thể chứng minh dơn giản như sau
Giả sử $f(x)$ là không tuyến tính giả sử $\deg{f(x)}=n$ khi đó $\deg{f(x)}\geq2$
thay vào hai vế của phương trình hàm ta thấy
+ bậc của vế phải là $2n$
+ Bậc của vế trái là $n$
Điều này dẫn tới sự mâu thuẫn từ đó tính duy nhất của hàm số $f(x)$ cũng được chứng minh.

"Bậc của hàm" khác với "bậc của đa thức" nhé bạn!
Hàm có thể không là một đa thức. VD: $f(x)=2^x$.
Thực ra bài gốc của bài này chỉ là tìm 2 hàm số thỏa mãn điều kiện trên. Còn tìm tất cả thì chủ quan mình thấy nó không khả quan.



#368736 Đề Thi Chọn HSG Vòng Trường - Tỉnh Thanh Hoá

Đã gửi bởi minhtuyb on 11-11-2012 - 16:26 trong Thi HSG cấp Tỉnh, Thành phố. Olympic 30-4. Đề thi và kiểm tra đội tuyển các cấp.

4.Cho $a,b,c$ thuộc $[1,3]$ và 4a+b+c=6$ . CMR: $a^2+b^2+c^2\leq 14$

Đề có nhầm không bạn?
$a,b,c\ge \Rightarrow 4a+b+c\ge 6$. Vậy từ gt suy ra $a=b=c=1$



#368729 Các ô vuông đc sắp xếp kề nhau tạo thành 1 dải hình chữ nhật vô hạn về cả 2 p...

Đã gửi bởi minhtuyb on 11-11-2012 - 16:11 trong Số học

Gợi ý: Nếu đánh số các ô lần lượt là $1,2,3,...,n$ và quy ước mỗi viên đá ở ô thứ $k$ mang giá trị $k^2$ thì tổng các giá trị $S$ của tất cả các viên đá là một đại lượng đơn biến (tăng thực sự)



#368725 Một túi gồm 1001 viên đá. Mỗi bước chọn 1 túi có nhiều đá hơn, bỏ đi 1 viên v...

Đã gửi bởi minhtuyb on 11-11-2012 - 16:05 trong Đại số

Một túi gồm 1001 viên đá. Mỗi bước chọn 1 túi có nhiều đá hơn, bỏ đi 1 viên và chia số còn lại thành 2 túi . Hỏi có thể làm như vậy để thu đc tất cả các túi đều có 3 viên đá đc không?

Gọi $S$ là tổng số đá và số túi sau mỗi bước $\Rightarrow S$ là đại lượng bất biến (sau mỗi bước bỏ 1 đá và thêm 1 túi)
Do lúc đầu $S\not\vdots 4$, mà để thu được tất cả các túi đều có 3 viên đá thì $S\vdots 4$
$\Rightarrow$ Không thể thu được tất cả các túi đều có 3 viên đá.



#368724 Cho 1 dãy số tự nhiên A(i) thỏa mãn: A(1) <2011, A (i) + A(i+1) = A(i+2)...

Đã gửi bởi minhtuyb on 11-11-2012 - 16:03 trong Số học

Cho 1 dãy số tự nhiên A(i) thỏa mãn:
A(1) <2011, A (i) + A(i+1) = A(i+2)

Biết A(1) - A(n) và A(2) + A(n-1) chia hết cho 2011.
CMR; N là 1 số lẻ

Xét dãy số $A_1=0;A_2=2011;A_i + A_{i+1} = A_{i+2}$ là một dãy số thoả mãn yêu cầu bài toán (do tất cả các số hạng của dãy đều chia hết cho $2011$)
Đâu nhất thiết là $n$ lẻ?



#367346 Một số bài toán tạo số và xếp người

Đã gửi bởi minhtuyb on 05-11-2012 - 22:02 trong Tổ hợp - Xác suất và thống kê - Số phức

Bài 3. Trên sân ga có đoàn tàu gồm 5 toa, 15 vị khách.

  • Có bao nhiêu cách xếp 15 vị khách lên tàu sao cho mỗi toa tàu có đúng 3 người.
  • Có bao nhiêu cách xếp 15 vị khách lên tàu sao cho một toa có 5 vị khách, một toa có 4 vị khách, một toa có 3, một toa có 2, một toa có 1 (vị khách)
___

Mọi người cùng thảo luận nhé !

*Ý 1: Xếp 15 vị khách thành một hàng. Cứ mỗi hoán vị của $15$ vị khách, ta có thể chia ra thành $5$ phần có $3$ vị khách.
Có tất cả $15!$ hoán vị của 15 vị khách.
Tuy nhiên khi ta thay đổi vị trí của các vị khách trên $1$ toa ta vẫn coi 2 cách sắp xếp đó giống nhau. Vậy đáp số là:$\dfrac{15!}{(3!)^5}$

*Ý 2: Ý tưởng tương tự ý 1. Đáp số là: $\dfrac{15!}{5!.4!.3!.2!.1!}$

Bây giờ ta thử mở rộng ý 1 bài toán trên:

Trên sân ga có đoàn tàu gồm 5 toa, 15 vị khách. Có bao nhiêu cách xếp 15 vị khách lên tàu sao cho một toa có 5 vị khách, một toa có 4 vị khách, một toa có 3, một toa có 2, một toa có 1 (vị khách), trong đó mỗi toa có một người làm "toa trưởng".



#366863 Chứng minh $AX,BY,CZ$ đồng quy

Đã gửi bởi minhtuyb on 03-11-2012 - 21:46 trong Hình học

Problem: Cho tam giác $ABC$ nội tiếp đường tròn $(O)$. $AD,BE,CF$ là đường kính của $(O)$. $(N_a),(N_b), (N_c)$ lần lượt là đường tròn $Euler$ của các tam giác $DBC,ECA,FAB$. Đường tròn $(K_a)$ tiếp xúc ngoài $(N_a)$ tại $X$ và tiếp xúc $AB,AC$ sao cho $K_a$ và $A$ khác phía $BC$. Tương tự có $Y,Z$.

Chứng minh rằng $AX,BY,CZ$ đồng quy.



#365644 Đề thi lập đội tuyển dự thi HSG Quốc gia THPT tỉnh Yên Bái (Vòng 1)

Đã gửi bởi minhtuyb on 28-10-2012 - 21:44 trong Thi HSG cấp Tỉnh, Thành phố. Olympic 30-4. Đề thi và kiểm tra đội tuyển các cấp.

Ngày thi: 22/10/2012

Thời gian: 180 phút

(Vòng 1)


Câu 1. (5 điểm)
Xác định tất cả các giá trị thực của m để từ đó tìm được 2 số nguyên a và b sao cho đa thức: $P(x)=x^5+mx-1$ chia hết cho đa thức: $Q(x)=x^2-ax+b$

Câu 2. (5 điểm)
Chứng minh rằng với mỗi số nguyên dương n cho trước thì phương trình:$x^{2n+1}=x+1$ có đúng một nghiệm thực. Gọi nghiệm ấy là $x_n$, tìm $\lim x_n$

Câu 3. (4 điểm)
Cho tam giác đều $ABC$ cạnh $a$, tâm $O$. Đường thẳng $d$ qua $O$ cắt $AB,BC,AC$ lần lượt tại $M, N, I$ . Chứng minh rằng: $\dfrac{1}{OM^2}+\dfrac{1}{ON^2}+\dfrac{1}{OI^2}$ không đổi khi $d$ quay quanh $O$.

Câu 4. (3 điểm)
Cho dãy số $(x_n)_{n=1}^7$ gồm các số nguyên dương thỏa mãn điều kiện:
$ x_6=144;\ \ \ x_{n+3}=x_{n+2}(x_{n+1}+x_n)$ với $n=1,2,3,4$.
Tính $x_7$

Câu 5. (3 điểm)
Cho bộ 3 số $(a;b;c)$ các số không đồng thời bằng nhau. Ta thực hiện phép toán sau: Nếu có bộ 3 số $(x;y;z)$ thì được thay thế bằng bộ 3 số $(x-y;y-z;z-x)$
a. Chứng minh rằng: Từ bộ 3 số $(a;b;c)$ ban đầu bằng cách thực hiện liên tiếp phép toán trên một số bước thích hợp ta nhận được bộ 3 số mà tồn tại ít nhất 1 trong 3 số đó lớn hơn số $22^{10}$.
b. Chứng minh rằng từ bộ 3 số $(a;b;c)$ ban đầu, với $a, b, c$ là các số nguyên. Nếu ta thực hiện liên tiếp phép toán trên một số bước thích hợp thì nhận được bộ 3 số mà ít nhất 1 trong 3 số đó chia hết cho $3^{2012}$



----------------------------- HẾT -----------------------------


Đã chém được $2,4,5$. Nhưng bài 4 làm vòng vèo quá ="=. Có đề mà không có đáp án so :(
P/s: Quê mình không hay cho BĐT, PTH thì phải? :ukliam2:



#365574 $n^4+2n^3+2n^2+2n+1=y^2$

Đã gửi bởi minhtuyb on 28-10-2012 - 17:42 trong Số học

Giải phương trình nghiệm nguyên $$n^4+2n^3+2n^2+2n+1=y^2$$

$$\Leftrightarrow (n^2+1)(n+1)^2=y^2$$
TH1: $n+1=y=0...$
TH2: $n+1,y\ne 0\Rightarrow n^2+1$ là số chính phương ...
Kết hợp 2TH có nghiệm: $(n;y)=(-1;0);(0;1);(0;-1)$

Bài này Toàn khúc mắc chỗ nào nhỉ?